PLAB MOCK Examination 1 With Answers And Explanations For The UK

PLAB MOCK Examination 1 With Answers And Explanations For The UK


PLAB MOCK Examination 1 With Answers And Explanations For The UK Abel  

4 years ago

~77.4 mins read

Mock1

Mock-june 2013.

1. A 56year old nulliparous woman with a BMI of 37 has a four--‐month history of blood stained discharge. She is otherwise normal. What is the single most likely diagnosis?
a. Atrophic Vaginitis
b. Bacterial Vaginosis
c. Endometrial Carcinoma
d. Candidiasis
e. Chlamydia

Ans. The key is C. Endometrial Carcinoma. [Post menopausal blood stained discharge is highly suggestive of endometrial carcinoma]. [a. Atrophic vaginitis- causes post coital bleeding, not blood stained discharge. B. Bacterial vaginosis- fishy odour (particularly postcoital odour) and increased vaginal discharge but bleeding is not a feature. d. Candidiasis- it causes vaginal erythema and curd-like or cheesy discharge, itching and dyspareunia, but bleeding is not a feature. e. Chlamydia- postcoital usually when there is ectropion. otherwise no bleeding]. 


2. A 23year old woman has been taking the combined oral contraceptive pill for 4 years. She has a troublesome clear, sticky discharge with no odour or itching. What is the single most likely diagnosis?
a. Cervical cancer
b. HIV Infection
c. Ectropion
d. Gonorrhea
e. Chlamydia

Ans. The key is C. Ectropion. [Oral contraceptive is a well known cause of cervical ectropion. There may be increase in vaginal discharge which is otherwise asymptomatic (no redness or plaque, no itching, no odour or other symptoms) in cervical ectropion].


3. A 25year old pregnant woman has a slight increase in vaginal discharge with vulval itching. Her partner has recent balanitis. What is the single most likely diagnosis?
a. Ectropion
b. Cervical polyp
c. Cervical carcinoma
d. Candidiasis
e. Atrophic Vaginitis

Ans. The key is D. Candidiasis. [In candidal vulvovaginitis there may be increased vaginal discharge which can be thin and watery or thick and white, like cottage cheese. There is vulval itching. Though it is not regarded as STD but is more common in sexually active patients and there may be candidal balanitis in male partner].

4. A 76year old woman has vaginal soreness and an intermittent pink staining on her underwear. She is not on any medication. What is the single most likely diagnosis?
a. Atrophic vaginitis
b. Bacterial vaginistis
c. Endometrial carcnoma
d. Candidiasis
e. Chlamydia

Ans. The key is A. Atrophic vaginitis. [Vaginal soreness in atrophic vaginitis may cause little discharge which may appear as pink staining on underwear].

5. A 19-year-old woman in a new sexual relationship has a vaginal discharge of sudden onset. There is intense itching at the introitus and examination reveals a frothy discharge with redness of her vulva. What is the single most likely diagnosis?
a. Syphilis
b. HIV Infection
c. HPV
d. Trichomonas Infection
e. Gonorrhea

Ans. The key is D. Trichomonas Infection. [Vaginal discharge with intense itching, redness of vulva and frothy discharge occur in trichomoniasis].

6. A 62year old woman has right-­â€sided headaches with malaise, weight loss and depression. What is the single most appropriate investigation?         
a. Temporal artery biopsy          
b. ESR         
c. Skull X-­â€Ray      
d. Nerve conduction         studies         
e. MRI imaging of brain.

Ans. The key is A. Temporal artery biopsy. [Right-sided (one sided) headache with malaise, weight loss and depression in 62 yrs elderly is highly suggestive of temoral arteritis for which “SINGLE” most appropriate investigation is temporal artery biopsy. If asked most appropriate answer would be ESR].

7. A 56-year-old man has sudden severe occipital headaches, drowsiness and vomiting. What is the single most initial appropriate investigation?
a. CT scan of brain
b. No investigation required
c. Carotid arteriography
d. Lumber puncture
e. Temporal artery biopsy

Ans. The key is A. CT scan of brain. [Sudden severe occipital headaches, drowsiness and vomiting are highly suggestive of SAH. Here single most initial appropriate investigation is CT scan of brain].

8. A 45-year-old woman has right sided headaches, nausea and visual disturbance. This lasted for 24 hours and she is fine. She was recently started on hormone replacement therapy. What is the single most initial appropriate investigation?
a. Temporal artery biopsy
b. ESR
c. No investigation
d. Nerve conduction studies
e. MRI imaging of the brain          
 
Ans. The key is C. No investigation. [One sided headache lasting for 24 hours associated with nausea and visual disturbance without any focal neurological sign is highly suggestive of migraine. So for this “no investigation is needed”].

9. A 46-year-old man has a headache and confusion three days after slipping in the garden. What is the single most appropriate investigation?
a. CT scan of the brain
b. No investigation required
c. Carotid arteriogram
d. Lumber puncture
e. Temporal artery biopsy
 
Ans. The key is A. CT scan of the brain. [Probable diagnosis is subdural hematoma. Slipping in the garden caused some injury to head which led to subdural hematoma supported by clinical features of headache and confusion after 3 days. So the investigation of choice in such case is CT scan of the brain].

10. A 58-year-old man presents with chest pain, which is aggravated by exercise but relieved with GTN spray. He has recurrent episodes of pain. Examination is normal and ECG shows new T wave depression in lead II, III and aVF. What is the single most initial combination to administer?
a. Antiplatelet drugs and low molecular weight heparin
b. Anti-platelet drugs and pravastin
c. Anti-platelet drugs and streptokinase
d. Low molecular weight heparin and warfarin
e. Low molecular weight heparin and pravastin
 
Ans. The key is A. Antiplatelet drugs and low molecular weight heparin. [This is a case of NSTEMI. So the single most initial combination to administer is a. Antiplatelet drugs and low molecular weight heparin. Low molecular weight heparin will prevent further events].

11. A 60-year-old woman has increasing frequency and urgency leading to occasional incontinence for 3 months. Abdominal and renal examination is normal. What is the most appropriate initial investigation?
a. Flexible cystoscopy
b. Retrograde cystourethrography
c. Ultrasound scan of the kidneys
d. Urinalysis for nitrates and leukocytes
e. Urodynamic studies
 
Ans. The key is E. Urodynamic studies. [Use of urodynamic studies: To know the causes of- incontinence, frequency, urgency, problems starting a urine stream, problem emptying bladder completely, UTI etc.].

12. A 30 year-old-man has developed a red raised lesion on his trunk and limbs after playing football one hour ago. He is feeling unwell with a slight cough. He had asthma, as a child but is otherwise well.

Advertisement

The rash is becoming increasingly itchy. What is the most initial appropriate treatment?
a. Intravenous adrenaline
b. Intravenous dexamethasone
c. Oral prednisolone
d. Oral chlorpheniramine
e. Intravenous penicillin

Ans. The key is D. Oral chlorpheniramine. [The patient developed generalized urticaria and he has no feature of anaphylaxis which needs adrenaline. This is treated with oral chlorpheiramine].

13. A 38-year-old woman has low mood, early morning awakening and poor appetite for six weeks. She had a similar episode 12 years ago and a further admission with over activity, irritability and grandiose. What is the single most likely diagnosis?
a. Post partum blues
b. Post partum depression
c. Bipolar affective disorder
d. Recurrent depressive disorder
e. Schizo-affective disorder
 
Ans. The key is C. Bipolar affective disorder. [The patient has now features of depression and 12 years ago she had similar symptoms and later symptoms of mania. So she is suffering from bipolar affective disorder].

14. A 47-year-old man has diurnal mood variation, early morning awakening and loss of interest and negative thoughts for four weeks. He has had no such previous episodes, he has been made redundant recently and as a result his house has been repossessed. What is the single most likely diagnosis?
a. Anxiety depressive disorder
b. Bipolar affective disorder
c. Major depressive disorder
d. Depression with personality disorder
e. Depression with psychotic symptoms

Ans. The key is C. Major depressive disorder.

15. A 32-year-old woman has poor concentration and forgetfulness for four weeks. She feels sad and tired with insomnia and poor appetite. What is the single most likely diagnosis?
a. Post partum blue
b. Post partum depression
c. Post partum psychosis
d. Recurrent depressive disorder
e. Depressive pseudodementia

Ans. The key is E. Depressive pseudodementia. [Depressive pseudodementia is a term commonly used to describe a condition whereby a patient experiences a cognitive deficit (here poor concentration and forgetfulness) secondary to a primary mood disorder].

16. A 29-year-old woman has had delusion of poverty and guilty ideas for some time. She has not had any such previous episodes. She is convinced that spies are after her and trying to kill her. She also admits to feeling lethargic. What is the single most likely diagnosis?
a. Anxiety depressive disorder
b. Bipolar affective disorder
c. Depressive personality disorder
d. Post partum blues

Ans. C. Depressive personality disorder. [Depression, and blaming and derogatory towards self, feeling guilty and remorseful along with persecutory delusion (which makes it paranoid type personality disorder) are suggestive of depressive personality disorder].

17. A 30-year-old lady has developed breast abscess 2 days after delivery of her second baby. What is the single most likely causative organism?
a. E. Coli
b. Group B beta hemolytic Streptococcus
c. Staphylococcus albus
d. Staphylococcus feacalis
e. Staphylococcus aureus
 
Ans. E. Staphylococcus aureus.

18. A 28-year-old lady developed her baby after 10 days feels that she is not good enough to take care of the baby and has a low mood, not eating well, exhaustion, inability to enjoy things that she previously enjoyed and sleep disturbance.
a. Post partum depression
b. Post partum psychosis
c. Major depressive disorder
d. Depressive personality disorder
e. Depression with psychotic symptoms
 
Ans. A. Post partum depression [If symptoms last longer then 2 weeks from birth or start later, patient could have postnatal depression. Postnatal depression can start any time in the first year after giving birth.

Signs that you or someone you know might be depressed include:
i. A persistent feeling of sadness and low mood
ii. Lack of enjoyment and loss of interest in the wider world
iii. Lack of energy and feeling tired all the time
iv. Trouble sleeping at night and feeling sleepy during the day
v. Difficulty bonding with your baby
vi. Withdrawing from contact with other people
vii. Problems concentrating and making decisions
viii. Frightening thoughts – for example, about hurting your baby].

19. A 57-year-old man recently underwent prostatectomy. Seven days later he develops pain and swelling of his left leg. This was later confirmed as deep venous thrombosis and he has to be started on warfarine. What is the optimal INR for this patient?
a. <1
b. 1 – 2
c. 2 – 3
d. 3 – 5
e. >5

Ans. C. 2 – 3

20. A 2-year-old boy has been unable to bear weight since the fall from a chair the previous night. He is tender in the mid tibia but there is no obvious deformity. What is the single most likely radiographic appearance?
a. Angulated fracture
b. Compound fracture
c. Epiphysial fracture
d. No trauma likely
e. Spiral fracture

Ans. E. Spiral fracture

21. An 8-year-old girl had recurrent central abdominal pains since 6 months. She also has occasional headaches. She maintains a normal appetite. There are no abdominal signs. FBC, urinalysis and stool microscopy are normal. What is the single most appropriate management?
a. Proton pump inhibitor
b. Pyeloplasty (+/-percutaneous nephrostomy)
c. Triple therapy
d. Reassurance
e. Sulphasalazine
 
Ans. D. Reassurance. [Given case is a case of abdominal migraine for which no specific treatment is recommended. Abdominal migraine is believed to be a variant of migraine that is common in children but rare in adults. Abdominal migraine is characterized by pain in the center of the abdomen that may be severe. Symptoms can last for one hour or up to several days. Nausea and vomiting may be associated with the pain. Abdominal migraine is treated by explanation and reassurance].

22. An 8-year-old boy has symptoms of right sided abdominal pain. He has had symptoms of UTI in the past. Abdominal ultrasound shows right-sided hydronephrosis and excretory isotope scan confirms PUJ obstruction. What is the single most appropriate management?
a. Pancreatic enzyme
b. Metoclopramide
c. Pyeloplasty (+/- percutaneous nephrostomy)
d. Enema and laxatives
E. Aspirin
 
Ans. C. Pyeloplasty (+/- percutaneous nephrostomy). [As there is obstruction of pelvi-ureteric junction it lead to hydronephrosis and it may need to relieve acute pressure by percutaneous nephrostomy. For cure we have to go through the procedure of pyeloplasty].

23. A 14-year-old boy has had recurrent abdominal pains, malaise and weight loss over 6 months. He has had a vague mass in the right iliac fossa. Colonoscopy and biopsy of the caecal area shows transmural inflammation and granulomata. What is the single most appropriate management?
a. Sulphasalazine
b.

Advertisement

Appendicectomy
c. Aspirin
d. Enema and laxatives
e. Gluten free diet
 
Ans. A. Salphasalazine. [Recurrent abdominal pains, weight loss and transmural inflammation and granulomata points towards the dx of Crohn’s disease which is treated with sulphasalazine].

24. An 8-year-old has had recurrent abdominal pain for 2 months. His appetite is reduced and he has faecal soiling. There are pitting masses throughout the abdomen. He has a dilated rectum. What is the single most appropriate management?
a. Proton pump inhibitor
b. Pyeloplasty (+/- percutaneous nephrostomy)
c. Enema and laxative
d. Aspirin
e. Triple therapy
 
Ans. Enema and laxative. [Pitting masses throughout the abdomen with dilated rectum indicates retained faecal matter. Enema and laxative are the treatment].

 25. A 12-year-old has recurrent abdominal pain which is worse when he is hungry. The pain is relieved by food or meal. He is tender in the epigastrium. Test has revealed Helicobacter pylori infection. What is the single most appropriate management?
a. Proton pump inhibitor
b. Enema and laxative
c. Aspirin
d. Gluten free diet
e. Metoclopramide
 
Ans. A. Proton pump inhibitor. [As patient has H. pylori infection the most appropriate management is triple therapy. But as it is not in the given options Proton pump inhibitor is the next answer].

26. A 26-year-old man presents to the A & E department with shortness of breath of 3 days duration. He has been coughing during the night and using his salbutamol inhaler more frequently. What is the single most appropriate investigation?

a. Arterial blood gases
b. Chest X-ray
c. Electrocardiogram (ECG)
d. Haemoglobin
e. Magnetic resonance imaging scan (MRI) of the chest
                                                 
Ans. A. Arterial blood gases. [Arterial blood gases are valuable for assessing severity of exacerbations and following response to treatment. It is useful to differentiate between type1 and type 2 respiratory failure].

27. A 22-year-old man presents to the A & E with sudden onset of sharp right-sided chest pain and shortness of breath. There is reduced air entry in the right side of the chest.
a. What is the single most appropriate investigation?
b. Peak expiratory flow
c. Pulse oxymetry
d. Spiral computed Tomography scan of the chest
e. Chest X-ray
f. Arterial blood gases
                                             
Ans. D. Chest X-ray. [Features (sudden chest pain with shortness of breath, reduced air entry) are consistent with pneumothorax for which Chest X-ray is the investigation of choice].

28. A 78-year-old woman is brought by her family to the A & E department with shortness of breath and occasional cough over the previous 72 hours. There is a coarse crepitation present at the right lung base. What is the single most  appropriate investigation?
a. Arterial blood gases
b. Chest X-ray
c. Electrocardiogram (ECG)
d. Haemoglobin
e. Magnetic resonance imaging scan (MRI) of the chest.
                                                     
Ans. B. Chest X-ray. [Probable pneumonia. Chest X-ray should be done].

29. A 26-year-old woman who has returned from a holyday in America develops sudden left sided chest pain and breathlessness. Her chest is clear. What is the single most appropriate investigation?
a. Peak expiratory flow
b. Pulse oxymetry
c. Spiral computed tomography (CT) scan pulmonary angiogram of the chest
d. Chest X-ray
e. Arterial blood gases
                                           
Ans. C. Spiral computed tomography (CT) scan pulmonary angiogram of the chest. [Probable pulmonary embolism which is a well-known complication of prolonged air travel and Spiral computed tomography (CT) scan pulmonary angiogram of the chest is done to establish the diagnosis].

30. A 23-year-old man with type 1 diabetes presents with 48 hours of increasing confusion and breathlessness. His chest is clear and his blood glucose concentration is 28 mmol/L. What is the single most appropriate investigation?
a. Chest X-ray
b. Arterial blood gas
c. Electrocardiogram (ECG)
d. Haemoglobn
e. Magnetic resonance imaging (MRI) scan of the chest.
                                            
Ans. B. Arterial blood gases. [Probabe DKA. In patients with DKA, arterial blood gases (ABGs) frequently show typical manifestations of metabolic acidosis, low bicarbonate, and low pH (less than 7.3)].

31. A 55-year-old man has weight loss, dyspnea and syncope. He smokes 20 cigarettes a day. Investigation confirms squamous cell carcinoma in the left bronchus. What is the single most important likely biochemical abnormality to be associated with the condition?
a. Hyperkalaemia
b. Hypercalcaemia
c. Hypernatremia
d. Hypocalcemia
e. Hypomagnesemia
                                              
Ans. B. Hypercalcaemia. [The main pathogenesis of hypercalcemia in malignancy is increased osteoclastic bone resorption. Enhanced bone resorption in malignancy is mainly secondary to different humoral factors released by tumour cells locally or systemically. The main humoral factor is parathyroid hormone related protein produced by many solid tumours. It increases calcium by activating parathyroid hormone receptors in tissue, which result in osteoclastic bone resorption. It also increases renal tubular resorption of calcium].

32. A 21-year-old woman in her 1st pregnancy has been admitted at 39 weeks gestation with breech presentation. She has had an eclamptic fit at home. She is conscious and observations are stable. What is the single most appropriate IV therapy to prevent further fits?
a. Chlomethiazole
b. Diazepam
c. Labetalol
d. Magnesium sulphate
e. Phenytoin
 
Ans. D. Magnesium sulphate. [Magnesium sulphate should be administered as infusions. If fits occurs loading dose of magnesium sulphate should be given. If fits recur magnesium sulphate bolus should be given till fits are controlled or side effects are evident (loss of reflexes or respiratory rate <14/min)].

33. A 14-year-old girl developed an itchy, scaly patch on her scalp. She has a similar patch that cleared spontaneously 2 years ago. Her aunt had similar undiagnosed rash on the extensor aspects of her elbow and knees. What is the single most likely diagnosis?
a. Eczema
b. Fungal infection
c. Impetigo
d. Lichen sclerosis
e. Psoriasis
                                      
Ans. E. Psoriasis. [Chronic plaque psoriasis is typified by itchy, well-demarcated circular-to-oval bright red/pink elevated lesions (plaques) with overlying white or silvery scale, distributed symmetrically over extensor body surfaces and the scalp].

34. A 58 year old man suddenly becomes shocked several days after suffering an acute myocardial infarction. His chest X-ray shows a large globular shaped heart and clear lung fields.  What is the single most likely diagnosis.
a. Acute pericarditis
b. Atrial thrombus
c. Cardiac tamponade
d. Heart block
e. Left ventricular aneurism
 
Ans. C. Cardiac tamponade. [Large globular heart indicates cardiac tamponade. Shock is due to compromised cardiac output due to tamponade].

35. A 63-year-old man continues to experience chest pain and has mild pyrexia two days after an acute myocardial infarction. His ECG shows widespread ST elevation and upward concavity. What is the single most likely diagnosis?
a. Left ventricular endocardial thrombus
b. Post myocardial infarction (Dressler’s syndrome)
c. Acute pericarditis
d. Ruptured inerventricular septum
e. Ruptured papillary muscle
                                                                
Ans. C. Acute pericarditis. [Chest pain, mild pyrexia and typical ECG findings (widespread ST elevation, upward concavity) indicates pericarditis. Dressler syndrome has similar pictute but occurs after 2-3 weeks of MI, not too early like 2-3 days].

36. A 48-year-old man has continuous anterior chest pain, worse on inspiration and pyrexia four weeks after a myocardial infarction. His ESR is raised. What is the single most likely diagnosis?
a. Left ventricular endocardial thrombus
b. Post myocardial infarction (Dressler’s Syndrome)
c. Acute pericarditis
d. Ruptured intraventricular septum
e. Ruptured papillary muscle
 
Ans. B. Post myocardial infarction (Dressler’s Syndrome). [Usually occurs 2-5 weeks after the initial event but it can be delayed for as long as 3 months. It is characterised by pleuritic chest pain, low-grade fever and pericarditis, which may be accompanied by pericardial effusion. It tends to follow a benign clinical course (though it may suggest further infarction). It is thought to be immune-mediated (antiheart antibodies may be present)].

37. A 60-year-old woman suddenly develops severe left ventricular failure following a myocardial infarction. Her echocardiogram demonstrates mild regurgitation. What is the single most likely diagnosis?
a. Left ventricular endocardial thrombus
b. Post myocardial infarction (Dressler’s syndrome)
c. Acute pericarditis
d. Ruptured intraventricular septum
e. Ruptured papillary muscle
 
Ans. E. Ruptured papillary muscle. [In ruptured papillary muscle there occurs mitral regurgitation].

38. A 55-year-old man returns for routine follow up 6 weeks after myocardial infarction. He gets breathless when walking uphill. His ECG shows ST elevation in the anterior chest leads. What is the single most likely complication?
a. Left ventricular endocardial thrombus
b. Post myocardial infarction (Dressler’s syndrome)
c. Acute pericarditis
d. Ruptured intraventricular septum
e. Left ventricular aneurysm

Ans. E. Left ventricular aneurysm. [Left ventricular aneurysm may occur following a heart attack. They usually arise from a patch of weakened tissue in a ventricular wall, which swells into a bubble filled with blood. This, in turn, may block the passageways leading out of the heart, leading to severely constricted blood flow to the body lowering stamina. Left ventricular aneurysm is frequently associated with ST elevation].

39. A 70-year-old man is brought to the A & E department by his daughter following overdose of 10 paracetamol tablets. His wife died 2 weeks ago. His only relevant past medical history was treatment for depression 10 years ago. His paracetamol level is below the treatment level. What is the most appropriate initial management?
a. Referral for social worker
b. Discharge home with advice
c. Refer for urgent psychiatric opinion
d. Refer for clinical psychiatric opinion

Ans. C. Refer for urgent psychiatric opinion. [Urgent psychiatric opinion is needed in psychiatric emergency where the person (patient himself) or person surrounding him is endangered by life threatening or destructive acts. In this patient if urgent intervention is not taken may overdose can happen again].

40. A 24-year-old woman fell and injured her right knee. The knee is swollen and tender to valgus stressing. Which of the following ligament is the most likely to have been damaged?
a. Anterior cruciate
b. Lateral collateral
c. Medial collateral
d. Oblique popliteal
e. Posterior
 
Ans. C. Medial collateral. [Positive valgus stress test indicate medial collateral rupture].

41. A 22-year-old man has had acute painful, red eye with blurring of vision for one day. He had similar episodes one year ago and has had back pain and stiffness relieved by exercise and diclofenac for years. What is the single most likely cause of the red eye?
a. Retinitis
b. Conjunctivitis
c. Iritis
d. Scleritis
e. Glaucoma
 
Ans. C. Iritis. [The young man has suggestive features of ankylosing spondylitis where iritis is a common association].

42. A 70-year-old man has back pain, anemia and weight loss of gradual onset. In addition to renal failure, he has hypercalcemia. What is the most appropriate investigation?
a. ANCA
b. Bone marrow aspirate
c. Creatinine clearance
d. Culture of MSU specimen
e. Cystoscopy

Ans. B. Bonemarrow aspirate. [Features are suggestive of multiple myeloma. Bone marrow aspirate will show plasma cell infiltrate (60% or more plasma cells in bone marrow)].

43. A 55-year-old man has leg swelling. He has heavy proteinuria and his serum creatinine concentration is 185mmol/L. What is the single most appropriate investigation?
a. IVU
b. Nuclear scan
c. Renal biopsy
d. Renal Ultrasound
e. Serum cholesterol concentration

Ans. C. Renal biopsy. [At this age the most common cause of nephrotic syndrome is focal glomerulosclerosis and then membranous glomerulonephritis for both of which renal biopsy is needed to establish the diagnosis].

44. A 75-year-old man has had a frequency of micturition, poor urinary stream and some incontinence for 2 years. His serum creatinine concentration is 350mmol/L. What is the single most appropriate investigation?

a. ANCA
b. Bone marrow aspirate
c. Creatinine clearance
d. Culture of MSU specimen
e. Cystoscopy
f. US of the kidney
 
Ans. F.

Admin Sponsored

Make money posting ads on your blog

Generate passive income with your blog, website or social media. Join Adsterra

Cinque Terre
US of the kidney. [features of prostatism is suggestive of prostate pathology and from the given options most appropriate is F. US of the kidney. Please note though US kidney is mentioned it is actually intended to mention US of KUB!! Diagnosis is BPH and US will show obstructive uropathy (like dilatation of ureter and calyx or hydronephrosis)].

45. A 65-year-old woman has urinary frequency, weight loss, hematuria and blood clots. An abdominal ultrasound is inconclusive. What is the single most appropriate investigation?
a. IVU
b. Nuclear scan
c. Renal Ultrasound
d. Serum cholesterol concentration
e. Cystoscopy
 
Ans. E. Cystoscopy. [Features are of bladder cancer supported by age, weight loss and hematuria. In bladder cancer there develop UTI like symptom as frequency, dysuria etc].

46. A 20-year-old woman with multiple sclerosis presents with blurring of vision and pain in the eye on movement. Where is the lesion?
a. Cerebellum
b. Optic discs
c. Optic nerve
d. Occipital
e. Retina
 
Ans. The key is C. Optic nerve. [Optic neuritis is a well known feature of multiple sclerosis. Described lesion is optic neuritis! Optic neuritis is a demyelinating inflammation of the optic nerve. It is also known as optic papillitis (when the head of the optic nerve is involved) and retrobulbar neuritis (when the posterior of the nerve is involved)].

47. A 65-year-old man after an open cholecystectomy experiences numbness around the umbilicus. Which nerve is damaged?
a. Diaphragmatic nerve
b. T4 nerve
c. Intercostal nerve
d. L1-L2 nerve root
e. T10 nerve root
 
Ans. The key is E. T10 nerve root. [Dermatom pattern of T10 nerve: Traverses posteriorly from T10 to anteriorly through the umbilicus].

48. A 30-year-old lady presents with sudden onset of right sided ilac fossa pain. She also experiences shoulder tip pain. What is the single most likely nerve damage?
a. Phrenic nerve
b. T12 nerve root
c. T5 nerve root
d. T10 nerve root
e. Intercostal nerve
 
Ans. The key is A. Phrenic nerve. [Phrenic nerve can be justified by referred pain to right shoulder but relation of right iliac fossa pain and phrenic nerve couldn’t be established!!]

49. A 25-year-old woman is 8 weeks pregnant. She has experienced a degree of vaginal bleeding and severe left sided abdominal pain. Her pulse rate is 100 beats/minute and her BP is 110/75 mmHg. There is marked cervical excitation on vaginal examination. What is the single most appropriate management?
a. Administration of anti-D
b. Bimanual vaginal examination
c. Blood transfusion
d. Colposcopy
e. Culdocentasis
 
Ans. The key is E. Culdocentasis. [In given case diagnostic laparoscopy should be the answer but as it is not in the options we should go for the most appropriate investigation from the given options which is E. Culdocentasis here. As patient is not in shock the dx is not ruptured ectopic pregnancy yet but leaking ectopic pregnancy in which there occurs accumulation of blood in pouch of douglas which can be revealed by culdocentasis].

50. An 18-year-old woman who is 13 weeks pregnant has a single episode of painless postcoital bleeding. The fetal heart is audible during the dopplar scan. What is the single most appropriate management?
a. Evacuation of retained products
b. Group and save serum
c. Laparoscopy
d. Monitor
e. Oral stilbesterol

Ans. The key is D. Monitor. [Painless postcoital bleeding gives rise to suspicion of placenta previa (cervical ectropion should be kept in mind also) for which at this stage we should monitor the patient].

51. A 28-year-old woman who is 10 weeks pregnant has had vaginal bleeding and period like pain for 5 days. Over the next 48 hours the symptoms have decreased. Examination is inconclusive. Beta HCG is very low and US scan reveals minimal uterine contents. What is the single most appropriate management?
a. Speculum examination
b. Ultrasound scan
c. Blood transfusion
d. Colposcopy
e. Evacuate product of conception

Ans. The key is E. Evacuate product of conception. [Diagnosis is incomplete abortion. Management is evacuation of product of conception].

52. A 28-year-old woman who is 9 weeks pregnant has a small amount of spontaneous vaginal bleeding. She has a mild vague lower abdominal pain. Her uterus is non-tender and equivalent for dates. What is the single most appropriate management?
a. Administration of Anti-D
b. Bi-manual vaginal examination
c. Blood transfusion
d. Culdocentesis
e. Transvaginal USS
                         
Ans. The key is E. Transvaginal USS. [As the uterus is equivalent for dates it is likely threatened abortion and transvaginal USS should be done to rule out complete or incomplete miscarriage and to see fetal heart beats].

53. A woman who is 10 weeks pregnant has heavy vaginal bleeding with marked uterine pain. Her pulse rate is 110 beats/min and BP is 110/75mmHg. Vaginal examination confirms that the blood is uterine and her cervix has begun to dilate.
a. Evacuation of retained products
b. Group and save serum
c. Laparoscopy
d. Monitor
e. Oral stilbesterol
                                      
Ans. The key is B. Group and save serum. [The diagnosis is inevitable abortion. 1st choice of management according to NICE is natural. So before taking further decision we can do blood grouping and save serum for the reference for next intervention].

54. A 23-year-old lady with multiple sclerosis presents with ataxia, dysarthria and nystagmus. Which of the following is the most likely location of the lesion?
a. Spinal cord
b. Brain stem
c. Cerebellum
d. Cerebral cortex
e. Medulla oblongata 

Ans. The key is C. Cerebellum. [Ataxia, dysarthria and nystagmus are well known features of cerebellar lesion].

55. A 45-year-old man presents with headache and vomiting. He is diagnosed with meningitis and a decition to perform a lumber puncture has been made. Which of the following structure will be pierced just before reaching the desired space?
a. Dura mater
b. Pia mater
c. Skin
d. Longitudinal muscles
e. Arachnoid matter
 
Ans. The key is E. Arachnoid matter. [The CSF occupies the subarachnoid space (between the arachnoid mater and the pia mater). So just before reaching this subarachnoid space we have to pierce the arachnoid matter].

56. A 40-year-old man presents with headache, vomiting and fever. He has been diagnosed with meningitis. Which of the following is a correct landmark to perform a lumber puncture?
a. L4
b. Iliac crest
c. Superior anterior iliac spine
d. Cox cocedeus
e. L5 vertebrea
 
Ans. The key is B. Iliac crest. [Lumber puncture is done in the L3-L4 interspace by palpating the right and left posterior superior spines of iliac crest and moving the fingers medially towards the spine].

57. A man was involved in a fight and sustained a blow to his mastoid bone. Which one is fractured?
a. Sphenoid bone
b. Mandibular bone
c. Temporal bone
d. Parietal bone
e. Occipital bone

Ans. The key is C. Temporal bone. [The temporal bone consists of four parts— the squamous, mastoid, petrous and tympanic parts].

58. A 35-year-old man is being investigated for infertility. Which of the following tests would prove that he has complete azoospermia?
a. Prolactin
b. Oestrogen level
c. Testosterone
d. Gonadotrophine
e. Follicle stimulating hormone

Ans. The key is C. Testosterone. [Low testosterone may lead to azoospermia. But surprisingly testosterone replacement can improve libido but not help azoospermia as it causes follicle stimulating hormone suppression. So testosterone rise should be done by natural means to improve fertility!]

59. A 70-year-old lady brings her husband who is 83 years of age. Which of the following additional factor will confirm the diagnosis?
a. Progressive functional disorder
b. Fluctuating functional disorder
c. Incontinence of urine
d. Suicidal intension
e. Stepwise deterioration
 
Ans. The key is A. Progressive functional disorder. [a. Alzheimer, b. Lewy body dementia, c. Normal pressure hydrocephalus, e. Vascular dementia].

60. A 5-year-old girl has had an upper respiratory tract infection (URTI) for 3 days and has been treated with paracetamol by her mother. The last 12 hours, she has been hot and irritable with severe pain in the right ear.
a. Acoustic neuroma
b. Chronic serous otitis media (glue ear)
c. Otitis media
d. Foreign body
e. Herpes zoster infection

Ans. The key is C. Otitis media.

61. A 38-year-old woman has just returned from a holyday where he went swimming every day. For the last few days he has irritation in both ears. Now his right ear is hot, red, swollen and acutely painful. What is the most likely diagnosis?
a. Impacted wax
b. Mumps
c. Otitis media
d. Otitis externa
e. Perforation of ear drum.

Ans. D. Otitis externa. [Presentation may prompt one to think otitis media, but in swimming there is only exposure of external ear to the water and no other risk factor for otitis media!! So it is not otitis media but otitis externa].

62. A 45-year-old man has a smooth, tender swelling extending from the ear to the angle of the jaw of sudden onset. He has a temperature of 38.5 C. What is the single  most likely diagnosis?
a. Presbycusis
b. Temporomandibular joint pain
c. Trigeminal neuralgia
d. Mumps
e. Otitis medisa

Ans. D. Mumps.

63. An 18-year-old man has a smooth, tender swelling from the ear to the angle of the jaw of sudden onset. He has a temperature of 38.5 C. What is the single most likely diagnosis?
a. Presbycusis
b. Temporomandibular joint pain
c. Trigeminal neuralgia
d. Mumps
e. Otitis media

Ans. D. Mumps.

64. A 75-year-old woman has weakness of left side of her face. She has had a painful ear for 48 hours. There are pastules in the left ear canal and on the eardrum. What is the single most likely diagnosis?
a. Impacted ear wax
b. Mumps
c. Otitis externa
d. Otitis media
e. Perforation of the eardrum
d. Ramsey Hunt syndrome

Ans. F. Ramsey Hunt Syndrome. [Herpes zoster oticus when presents with facial nerve involvement is known as Ramsey Hunt syndrome].

65. A 45-year-old woman has her left ear syringed 48 hours ago. During this procedure she had acute pain. Now she has discharge and deafness. Which vein is likely to be involved?
a. Perforated eardrum
b. Azygous vein
c. Bracheocephalic vein
d. Cephalic vein
e. External iliac vein
 
Ans. Perforated eardrum!! (Inappropriate question).

66. A 43-year-old has pain and stiffness to her leg. The limb is swollen and cyanosed from the upper thigh to her ankle. A large uterine mass is palpable. Which vein is likely to be involved?
a. Inferior vena cava
b. Internal iliac vein
c. External iliac vein
d. Long sephanous vein
e. Median cubital vein 

Ans. C. External iliac vein. [external iliac vein is the deep vein of lower limb and compression of it can result in chronic leg pain and swelling along with cyanosis].

67. A 41-year-old woman has varicose vein on the lateral side of her right lower leg. The vein is tortuous and dilated. Which vein is likely to be involved?
a. Popliteal vein
b. Profunda femoris vein
c. Renal vein
d. Short sephanous vein
e. Axillary vein 

Ans. D. Short sephanous vein.

Advertisement

[Short sephanous vein courses through lateral side of the leg while long or great sephanous vein courses medially].

68. A 25-year-old man has a varicosity on his lower abdominal wall. There is a small testicular mass which is subsequently found to be malignant. Which vein is likely to be involved?
a. Axillary vein
b. Azygous vein
c. Bracheocephalic vein
d. Renal vein
e. External iliac vein

Ans.  D. Renal vein.

69. A 60-year-old has pain and stiffness of his left leg. His calf is swollen and feels hard. Which vein is likely to be involved?
a. Inferior vena cava
b. Internal iliac vein
c. Internal jugular vein
d. Long sephanous vein
e. Popliteal vein
 
Ans E. popliteal vein.

70. A 28-year-old man who has spent his weekend decorating his house has swelling and discomfort in his right arm of sudden onset. The upper limb is swollen, congested and cyanosed. Which vein is involved?
a. Popliteal vein
b. Profunda femoris vein
c. Renal vein
d. Short sephanous vein
e. Axillary vein

Ans. E. Axillary vein.

71. A 30-year-old lady presents with foul smelling discharge. Endocervical swab shows trichomonas. Which of the following is the most appropriate treatment?
a. Amoxicillin
b. Metronidazole
c. C0-trimoxazole
d. Tetracycline
e. Ceftriaxone

Ans. B. Metronidazole. [Metronidazole is very effective if taken correctly. One should have to take metronidazole twice a day, for five to seven days. Ref: NHS].

72. A 69-year-old man with heart failure develops gout. He has recently been started on treatment and his heart failure worsens. Which of the following is the most likely cause?
a. NSAIDs
b. B-blockers
c. Bendroflumethiazide
d. Calcium channel blocker
e. Allopurinol
                                       
Ans. A. NSAIDs. [NSAIDs used for gout worsens heart failure by retention of salt and water].

73. A 7-year-old 7 days post tonsillectomy presents with nose bleed. He is currently not bleeding. What is the most appropriate treatment?
a. Apply pressure on the soft part
b. Antibiotics
c. Reassure
d. Surgical exploration
e. Local adrenaline
                                      
Ans. B. Antibiotics. [Bleeding from post operative site may occur from infection which causes necrosis and sloughing out of dead tissue. So such bleeding is treated by antibiotics to control risk of further bleeding].

74. A 3-year-old boy has a sudden onset of cough, shortness of breath and noisy breathing. His mother reports that this is followed by a choking episode while he was playing.
a. Cystic fibrosis
b. Epiglottitis
c. Immunodeficiency
d. Inhaled foreign body
e. Recurrent aspiration

Ans. D. Inhaled foreign body. [During playing with toy, toy or part of it can enter respiratory tract of young children and cause above mentioned symptoms].

75. A 2-year-old boy has repeated chest infections, offensive loose stools and recurrent rectal prolapse.
a. Tracheomalacia
b. Tuberculosis
c. Asthma
d. Cystic fibrosis
e. Bronchiectasis

Ans. D. Cystic fibrosis. [Repeated chest infections, offensive stools, (stress incontinence) and rectal prolapse are recognized features of cystic fibrosis].

76. A 4-year-old girl who has had a runny nose for 12 hours has woken up 2’oclock in the morning with difficulty in breathing, a barking cough. What is the single most likely diagnosis?
a. Asthma
b. Bronchiectasis
c. Bronchiolitis
d. Bronchopulmonary dysplasia
e. Croup
 
Ans. E. Croup. [Age, runny nose, dyspnea, and barking cough are suggestive features of croup].

77. A 4-year-old girl whose parent have refused to allow immunizations has a fever, difficulty in breathing, stridor, drooling and central cyanosis. What is the single most likely diagnosis?
a. Cystic fibrosis
b. Epiglottitis
c. Immunodeficiency
d. Inhaled foreign body
e. Recurrent aspiration
 
Ans. B. Epiglottitis. [Haemophilus influenza, Haemophilus parainfluenzae, Streptococcus pneumoniae, and group A streptococci are the common causes of epiglottitis].

78. A 16-year-old footballer presents with sudden onset testicular pain, which started an hour ago. What is the most likely diagnosis?
a. Trauma
b. Testicular torsion
c. Epididymo-orchitis
d. Varicocele
e. Hernia
 
Ans. B. Testicuar torsion. [Sudden onset, no fever is more likely to be testicular torsion].

79. A 70-year-old man who is on bendroflumethiazide develops an acute swelling of his right knee. What is the single most appropriate diagnostic investigation?
a. Serum uric acid
b. Reactive protein
c. D-dimer
d. Blood culture
e. Plain X-ray of the knee

Ans. A. Serum uric acid. [Bendroflumethiazide can cause hyperurecemia leading to gout].

80. A 45-year-old man with diabetes mellitus has a high ferritin. What type of cancer is the most likely to develop?
a. Liver cancer
b. Lung cancer
c. Renal cancer
d. Colonic cancer
e. Gastric cancer
 
Ans. A. Liver cancer. [In patient with DM ferritin level is higher than control. This ferritin is indicative of high iron level in liver ultimately leading to liver cancer].

81. A 70-year-old man had a painful swelling of his right testis for 36 hours. He has a history of dysuria and frequency of micturition and has a temperature of 38.C
a. Epididymal cyst
b. Epididymo-orchitis
c. Hydrocele
d. Inguinal hernia
e. Scrotal abscess
 
Ans. B. Epididymo-orchitis. [Painful swelling of testis with features of UTI and raised temperature makes the likely diagnosis to be Epididymo-orchitis].

82. A 25-year-old man has had painless enlargement of his left testis for 3 months. The left testis is 3 times the size of right testis and is non-tender. What is the single most likely diagnosis?
a. Sebacious cyst of the scrotunm
b. Testicular hematoma
c. Testicular neoplasm
d. Hydrocele
e. Epididymal cyst
 
Ans. D. Hydrocele. [Though testicular neoplasm is also possible but hydrocele is much more common than tumour].

83. A 17-year-old lady presents to the A&E department after taking 45 tablets of paracetamol following an argument with her boyfriend. The incident happened 10 hours ago. What is the single most appropriate next action?
a. Activated charcoal
b. Check paracetamol level
c. N-acetylcysteine
d. Observe
e. Do blood tests after 4 hours
 
Ans. C. N-acetylcisteine. [If >8–24h and suspicion of large overdose (>7.5g) err on the side of caution and start N-acetylcysteine, stopping it if level below treatment line and INR/ALT normal]. Ref: OHCM, 9th edition, page-856.

84. A 56-year-old man with known HIV presents with chronic diarrhea. What is the most likely cause?
a. Cryptococcus infection
b. Toxoplasmosis
c. Pneumocystis carinii
d. Cryptosporidium
e. Herpes zoster
                                               
Ans. D. Cryptosporidium. [In AIDS patients common cause of diarrhea is cryptosporidium].

85. A 65-year-old man who is an intravenous drug abuser presents with weight loss and maculopapular rash. What is the single most likely cause?
a. Cryptococcus infection
b. HHV8
c. Pneumocystis carinii
d. Human papilloma virus
e. Cryptosporidium
 
Ans. B. HHV8. [human herpes virus 8 or, kaposis sarcoma associated herpes virus].

86. A 42-year-old woman has not had a period for 10 months. She feels well but she has hot flushes. Her mother also had an early menopause.
a. Cervical smear
b. Chest-x-ray
c. Dexa scan
d. FSH/LH
 
Ans. D. FSH/LH. [FSH and LH are raised in premature ovarian failure; an FSH level ≥20 IU/l in a woman aged under 40 with secondary amenorrhoea indicates premature ovarian failure].

87. A 60-year-old woman under long term follow up after a lumpectomy and therapy for breast cancer at the age of 50 comes to discuss her bone health following a stress fracture to her metatarsal bone.
a. Lumber spine X-ray
b. Dexa scan
c. Mid stream urine culture
d. No investigation
e. Serum calcium concentration

Ans. B. Dexa scan. [60 yrs of age female are prone to osteoporosis due to reduced oestrogen and dexa-scan is used in susceptible patients].

88. A 36-year-old lady who is an intravenous drug user has had weight loss for the past 4 months. He now presents with drowsiness and increasing confusion. What is the single most likely cause?
a. Cryptococcus infection
b. Toxoplasmosis
c. Pneumocystis carinii
d. Cryptosporidium
e. Herpes zoster

Ans. The key is A. Cryptococcus infection. This is a wrong key!! Correct key is B. Toxoplasmosis. [Case of HIV. Cryptococcus usually causes meningitis in AIDS patients and in meningitis mental clearance is usually maintained! That is there does not occur confusion. On the other hand cerebral toxoplasmosis is the infestation where drowsiness and confusion occurs].

89. A 36-year-old woman is complaining of mood swings, palpitations and irregular menses.
a. Transvaginal USS
b. FSH
c. Chest X-ray
d. Dexa scan
e. ECG
 
Ans. B. FSH. [Symptoms are of perimenopause and in this patients likely indicates going to be premature ovarian failure].

90. A 20-year-old mother brings her 2 year old child and she is worried that her child be deaf as she is deaf herself. What is the single most appropriate investigation?
a. Audiometry
b. Tuning fork
c. Audiometry brain stimulating
d. Reassure
e. Distraction test

Ans. The key is A. Audiometry.

91. A 75-year-old man is having a laparoscopic cholecystectomy. 2 ports on the lateral abdomen have been inserted. When placing the 3rd port, which structure will be damaged?
a. Linea alba
b. Latisimus dorsi
c. Pectolaris major
d. Skin
e. Intercostal muscle

Ans. The key is A. Linea alba.

92. A 21-year-old student has several episodes of palpitations, sweating, nausea and overwhelming fear of sudden onset. On one occasion, she has woken from sleep in this state and fears she is going insane. There is no previous history of any psychiatric disorder. What is the single most likely diagnosis?
a. Adjustment disorder
b. Agoraphobia
c. Anorexia nervosa
d. Bipolar disorder
e. Panic attack
 
Ans. The key is E. Panic attack.

Advertisement

[Panic attack can be frightening and happen suddenly, often for no clear reason with following features, palpitations, sweating, trembling, hyperventilation, nausea, and few more].

93. A 40-year-old businessman is worried that his alcohol consumption has reached dangerous levels. He has recently changed jobs, which involve travelling and corporate hospitality. He finds it increasingly difficult to give presentations to new business associates and finds that alcohol helps him to function.
a. Major depressive episode
b. Obsessive compulsive disorder
c. Panic disorder
d. Post traumatic stress disorder
e. Social anxiety disorder
 
Ans. The key is E. Social anxiety disorder.

94. A 32-year-old mother is in tears, complaining of constant irritability with her 2 small children and inability to relax. She describes herself as easily startled with poor sleep disturbed by nightmares, following a serious house fire one year ago while the family slept.
a. Post traumatic stress disorder
b. Adjustment disorder
c. Agoraphobia
d. Anorexia nervosa
e. Bipolar disorder
 
Ans. The key is A. Post traumatic stress disorder. [  Post-traumatic stress disorder (PTSD) is an anxiety disorder caused by very stressful, frightening or distressing events. Someone with PTSD often relives the traumatic event through nightmares and flashbacks. They may also have problems sleeping, such as insomnia, and find concentrating difficult].

95. A 49-year-old man has returned home 6 times daily from work for the past many years worried that he has left the taps running. His employers are concerned about his poor work productivity.
a. Adjustment disorder
b. Agoraphobia
c. Anorexia nervosa
d. Obsessive compulsive disorder
e. Generalized anxiety disorder

Ans. The key is D. Obsessive compulsive disorder. [Obsessive-compulsive disorder (OCD) is characterized by unreasonable thoughts and fears (obsessions) that lead one to do repetitive behaviors (compulsions)].

96. A 40-year-old woman has acute symptoms of palpitations, sweating, difficulty in breathing and feeling unsteady on her feet. She has noticed her symptoms occur only in crowded public places.
a. Major depressive disorder
b. Obsessive compulsive disorder
c. Agoraphobia
d. Post traumatic stress disorder
e. Social anxiety disorder
 
Ans. C. Agoraphobia. [One may have agoraphobia if he may has fear of:
Entering shops, crowds, and public places.
Travelling in trains, buses, or planes.
i. Being on a bridge.
ii. Being in a lift.
iii. Being in a cinema, restaurant, etc, where there is no easy exit.
iv. Being anywhere far from your home].

97. A 75-year-old man has an episode of syncope while working in the garden. When visited by his doctor 30 minutes later, he is in sinus rhythm; his pulse rate is 74 bpm. His blood pressure is 110/60 mmHg and he has a systolic murmur. What is the single investigation most likely to provide the definitive diagnosis?
a. Coronary angiogram
b. Echocardiogram
c. Electrocardiogram
d. 24 hour ambulatory blood pressure
e. 24 hour taped ECG
                                                                                                                  
Ans. B. Echocardiogram. [Probable diagnosis is aortic stenosis which can cause syncope during exercise particularly in elderly person which can be diagnosed by echocardiogram].

98. An 83-year-old-woman who is resident in a nursing home is admitted to hospital with a four day history of diarrhea. She has had no weight loss or change in appetite. She has been on analgesics for 3 weeks for her back pain. She is in obvious discomfort. On rectal examination, she is found to have fecal impaction. What is the single most appropriate immediate management?
a. Codeine phosphate for pain relief
b. High fibre diet
c. Oral laxative
d. Phosphate enema
e. Urinary catheterization

Ans. The key is D. Phosphate enema. [This diarrhea is overflow diarrhea of fecal impaction and phosphate enema will help to bring out feacal mass and resolve the condition].
  
99. A 65-year-old man has a productive cough and fever. He has influenza and his temperature is 38.5 C. There is reduced air entry and crepitations bilaterally. What is the single most likely cause of the pneumonia?
a. Haemophilas influenza
b. Klebsiella pneumonia
c. Mycoplasma pneumonia
d. Staphylococcus aureus
e. Streptococcus pneumonia

Ans. The key is D. Staphylococcus aureus. [Pneumonia following influenza is frequently due to staphylococcus aureus with bilateral crepitations and with productive cough and fever].

100. A 67-year-old lady has lung cancer and she suffers from heartburn, which increases at night. She has tooth carries and has offensive mouth odour. She can not cope with it. What is the single most appropriate treatment for her?
a. Erythromycin
b. Vancomycin
c. Metronidazole
d. Proton pump inhibition
e. Anty H. pylori treatment

Ans. The key is C. Metronidazole. This is a wrong key. Correct key is D. Proton pump inhibitor. [Cancer itself does not cause halitosis (though lung cancer can produce halitosis but not treated by metronidazole unless there are signs of infection) but secondary to chemotherapy as its side-effects. Metronidazole is not the treatment of chemotherapy halitosis. This patient has clear picture of GERD (tooth carries are due to acid reflux) and it is a leading cause of halitosis. Single most appropriate treatment for GERD induced halitosis is proton pump inhibitor].

101. A 20-year-old man has a head on collision in a car. He is not breathless and has chest pain as he fractured his 5 – 7th ribs. Chest X-ray confirms this fractures. What is the most appropriate initial action?
a. Antibiotics
b. Analgesia
c. Oxygen mask
d. Physiotherapy
e. Refer to surgeon

Ans. The key is B. Analgesia.  [Most appropriate initial action in flial chest is oxygen mask if the patient is breathless! But as the case is not breathless and having pain most appropriate initial action is analgesia].

102. A 54-year-old man comes with a sudden onset of palpitation and breathlessness. His heart rate is 164 beats/minute. What is the single most appropriate treatment in acute phase?
a. Adenosine
b. Metoprolol
c. Verapamil
d. Amiodarone

Ans. The key is C. Verapamil. [Though it can be thought as SVT but SVT has no acute or chronic phase!! So the Dx is given in the question and it is acute phase of the arrhythmia; i.e. atrial fibrillation (which has acute and chronic phase). Acute phase if hemodynamically stable (as in presenting case: no feature of hemodynamical unstability is described) 1st line treatment is verapamil. Ref: OHCM].

103. A young adult presents to the A&E after a motorcycle crush. The patient has bruises around his left orbital area. His GCS is 13 on examination and a strong alcoholic breath is noticed. Shortly afterwards, His GCS drops to 7. What is the single most important  initial assessment?
a. MRI brain
b. CT brain
c. Chest X-ray
d. CT angiogram brain

Ans. The key is B. CT brain.

Advertisement

[Young adult with considerable head injury (like RTA, fall from hight etc.) with drop of GCS indicates acute extradural hematoma for which CT brain is the investigation of choice].

104. A 27-year-old woman at 34 weeks gestation in her first pregnancy attends ANC. Her blood results showed Hb. 10.6, MCV: 95, MCHC: 350. What should you do for her?
a. Folate
b. Dextran
c. Ferrous sulphate
d. None

Ans. The key is C. Ferrous sulphate. This is a wrong key! Correct key is D. None. [According to NICE, cut offs for iron supplements:
at booking (8-10 weeks)- if less than 11
at 28 weeks and further- if less than 10.5
if less than these values=> give iron].

105. A 60-year-old man is on treatment for ischemic heart disease and hypertension and hyperlipidemia. During the night, he complains of wheezing and shortness of breath. Which of the following medication is responsible for his symptoms?
a. Amlodipine
b. Atenolol
c. Ramipril
d. Simvastatin
e. Bendroflumethiazide
 
Ans. The key is D. Simvastatin. [Shortness of breath is found among people who take Simvastatin, especially for people who are female, 60+ old , have been taking the drug for < 1 month, also take medication Aspirin, and have High blood cholesterol]. [On the other hand Atenolol though occasionally can cause bronchoconstriction in a few susceptible patient but as cardioselective usually does not cause bronchoconstriction].

106. A 30-year-old man complains of pain in the loin with a blood pressure of 140/90 mmHg. He is found to have proteinuria and hematuria. What is the investigation would you do to confirm the diagnosis?
a. Ultrasound of abdomen
b. ANCA
c. ANA
d. Urine microscopy and culture
e. Stool culture
 
Ans. The key is A. Ultrasound of abdomen. [Dx is glomerulonephritis. Ultrasound will detect kidney size to determine whether kidney sizes are normal (acute glomerulonephritis) or reduced (chronic glomerulonephritis)].
 
107. A 4-year-old girl is taken by her mother to the A & E with feeling of being unwell, urinary urgency and 39.C temperature. What is the single next best investigation?
a. Catheter catch of urine
b. Clean catch of urine
c. Ultrasound
d. Intravenous urography (IVU)
e. Suprapubic catch of urine

Ans. The key is B. Clean catch of urine. [Unwell, urgency and fever are suggestive of UTI for which next best investigation is clean catch of urine].

108. A 35-year-old lady comes with a 5-year-old child who has nystagmus, tinnitus and deafness. The boy is found to have tumour in the cerebellopontine angle. On examination, the boy is found to have numerous café-au-lait spots seen on the back. What is the inheritance ratio of this condition?
a. 100%
b. 1:2
c. 1:4
d. 1:8
e. None

Ans. The key is B. 1:2. [Probable diagnosis is neurofibromatosis type 2. Mode of inheritance is autosomal dominant. So the inheritance ratio is 1:2].

109. A 70 year old man with cerebral metastasis develops cerebral oedema and is being given dexamethasone and antihypertensive drugs. His blood glucose rises to 17-18 mmol/l. What is the appropriate management?
a. Low dose insulin
b. Optimal dose of insulin
c. Glibenclamide
d. Stop dexamethasone

Ans. The key is A. Low dose insulin. [Dexamethason induced hyperglycemia is not usually controlled tightly but with low dose insulin. Preferred method is night time basal dose (like with glargine) at bed time and premeal short acting insulin prior to each meal like lispro].

110. A 53-year-old female presents with an acute painful hot knee joint. She is known case of rheumatoid arthritis. On examination, the skin is red, tender and swollen. The hamstring muscles are in spasm. Her temperature is 38.5.C and blood pressure is 120/80 mmHg. What is the single best next investigation?
a. Joint aspiration for cytology, culture and sensitivity
b. Joint aspiration for positively birefringent crystals
c. Joint aspiration for negatively birefringent crystals
d. Blood culture
e. Serum uric acid

Ans. The key is A. Joint aspiration for cytology, culture and sensitivity. [Acute painful hot knee joint is suggestive of septic arthritis. Also underlying rheumatological disease like ra are more prone to septic arthritis. So the single best next investigation is joint aspiration for cytology, culture and sensitivity].

111. A 75-year-old man complaining of dripping incontinence and has an ammonia smell. On examination, He has a furred tongue and a painless abdominal mass with extension to the umbilical level. Urine analysis shows proteins ++ and nitrates + with no growth. What is the most appropriate management for this patient?
a. Suprapubic catheterization
b. Trimethoprim
c. Intraurethral catheterization
d. Laparotomy
e. Condom catheter

Ans. C. Intraurethral catheterization. [Patient’s urine has no growth! So no need for antibiotics. As there is overflow incontinence and bladder remains full, condom catheter is not suitable but intraurethral catheter].

112. What is the most appropriate antibiotic to treat uncomplicated chlamydia infection in a 21-year-old female who is not pregnant?
a. Erythromycin
b. Ciprofloxacin
c. Metronidazole
d. Cefixime
e. Doxycycline

Ans. The key is E. Doxycycline.

113. A child complains of right iliac fossa pain and diarrhea. On colonoscopy granular transmural ulcers are seen near the iliocecal junction.
a. Sulfasalazine
b. Paracetamol
c. Ibuprofen
d. Metronidazole
e. Steroid

Ans. The key is A. Sulfasalazine. This is wrong key! Correct key is E. Steroid. [This is a case of Crohn’s disease. In Crohn’s disease Steroids are 1st line. Ref: NHS.uk]
  
114. A 42-year-old man has been tired and sleepy for the last few weeks in the morning. His work has started being affected as he feels sleepy in the meeting. His BMI is 36kg/m2. What is the single most likely diagnosis? 
a. Idiopathic narcolepsy
b. Narcolepsy
c. Chest hyperventilation syndrome
d. Obstructive sleep apnoea syndrome
e. REM related sleep disorder

Ans. The key is D. Obstructive sleep apnoea syndrome. [Daytime somnolence and high BMI]. 

115. A 47-year-old man with a history of ischaemic heart disease complains of chest pain with shortness of breath on exertion over the past few days. ECG is normal and echocardiography shows decreased ejection fraction and decreased septal wall thickening. What is the single most likely diagnosis?
a. Dilated cardiomyopathy
b. Constrictive pericarditis
c. Amyloidosis
d. Subacute endocarditis

Ans. The key is A. Dilated cardiomyopathy. [Diagnosis is ischemic dilated cardiomyopathy].

116. A 76 year old woman on paroxetine has increasingly become confused and weak. What is the single most likely reason for her confusion?
a. Hypercalcemia
b. Hypernatremia
c. Hypocalcemia
d. Hypoglycemia
e. Hyponatremia

Ans. The key is E. Hyponatremia. [A well known complication of paroxetine therapy is SIADH causing hyponatremia. Therefore we should be alert to hyponatremia in patients on paroxetine by carrying out periodic monitoring of serum electrolytes, especially in elderly patients].

117. A 46-year-old man is being investigated for indigestion, jejunal biopsy shows deposition of macrophages containing PAS positive granules. What is the most likely diagnosis?
a. Bacterial overgrowth
b. Celiac disease
c. Tropical sprue
d. Whipple’s disease
e. Small bowel lymphoma

Ans. The key is D. Whipple’s disease. [Macrophages containing PAS positive granules on jejunal biopsy is diagnostic of Whipple’s disease].

118. A 30-year-old woman has IUCD inserted 8-9 months ago. Now on routine follow-up the thread is missing. Uterine ultrasound shows no IUCD in the uterus. What is the best management?
a. Laparoscopy
b. Pelvic CT scan
c. Laparotomy
d. Pelvic X-ray
e. MRI

Ans. The Key is D. Pelvic X-ray. [Management of Lost Thread: Ultrasound should be arranged to locate the device. If ultrasound does not locate the device and there is no definite history of expulsion then abdominal X-ray should be performed to look for an extrauterine device. Expulsion should not otherwise be assumed. Hysteroscopy can be helpful if ultrasound is equivocal. Surgical retrieval of an extrauterine device is advised].

119. A man with dementia has an ulcerative lesion on his forehead. He wants it to be removed so that “it can improve his memory.” The wife says he is not fit to give consent. What will you do?
a. Get the GP to sign the consent
b. Get the wife to sign the consent
c. Get the patient to sign the consent
d. Refer to psychiatrist to assess the mental capacity to give consent

Ans. The key is D. Refer to psychiatrist to assess the mental capacity to give consent.

120. A 6-year-old-boy is brought to the A&E with pain in his left arm following a fall from a tree on his outstretched hand. There is a swelling around his left elbow and a reduced radial pulse. What is the most likely diagnosis?
a. Angulated fracture
b. Compound fracture
c. Epiphyseal fracture
d. No fracture
e. Spiral fracture

Ans. The key is Given keys have not the correct answer!! The correct answer is Supracondylar fracture.

121. A young woman fainted in the street and was brought the A&E. She weighs 33 kg. She is alert and well in the A&E and states she is on a diet and admits to not eating or drinking anything for 48 hours. She is annoyed that she has been brought to the hospital and adamantly states that she wants to leave. What is the most appropriate next step? 
a. Detain her under the Mental Health Act
b. Refer to the dietician
c. Allow her to be discharged
d. Refer her to the GP

Ans. The key is A. Detain her under the Mental Health Act.

122. A woman with mild ulcerative colitis is not on any treatment. She develops diarrhea and mucous in the last three months. All other examinations are normal. Her Hb is 9.6 gm/dl. What is the single best treatment?
a. Magnesium sulphate
b. Ferrous sulphate
c. Meselazine
d. Endoscopy
e. Colonoscopy

Ans. The key is C. Meselazine. [Mesalazine - 5-aminosalicylic acid (5-ASA) - is now the treatment of choice for induction and maintenance of remission of mild-to-moderate ulcerative colitis. Oral mesalazine is less effective than oral corticosteroids and so should be used as sole treatment only in mild attacks. Topical mesalazine is probably slightly more effective than topical corticosteroids. Ref: patient.info].

123. A smoker with several ulcers in the mouth and lower surface of the tongue and also with white striae on the mouth.
a. Aphthous ulcer
b. Lichen planus
c. Kaposis sarcoma
d. Cancer of the mouth

Ans. The key is D. Cancer of the  mouth. [Smoking is a risk factor for cancer of mouth and oral ulcers are common presenting feature of mouth cancer].

124. A middle aged man complains of a node which has been growing for several months. Now it is firm and centrally depressed. What is the most likely diagnosis?
a. Basal cell carcinoma
b. Squamous cell carcinoma
c. Pyoderma gangrenosum
d. Melanoma
e. Necrobiosis lipoidica
 
Ans. The key is A. Basal cell carcinoma.

125. A 76-year-old man presents with loss of sensation on the middle part of the chin and lower lip. Which nerve is involved?
a. Lingual
b. Inferior alveolar
c. Auricular
d. Submental
e. Buccal nerve.

Ans. The key is D. Submental.
 
126. A 37-year-old man had hemicolectomy. Immediately after operation, he becomes aggressive not allowing doctors to give him analgesia. 2 days later he thanked the doctor.
a. Delirium
b. Acute stress reaction
c. Depression
d. Adjustment disorder
e. Alcohol withdrawal

Ans. The key is B. Acute stress reaction. [Acute stress reaction (also called acute stress disorder, psychological shock, mental shock, or simply shock) is a psychological condition arising in response to a terrifying or traumatic event (here surgery), or witnessing a traumatic event that induces a strong emotional response within the individual].

127. A 60-year-old lady has had breast surgery done for breast cancer.  Which one of the following will increase the chance of recurrence?
a. Sensitivity to estrogen receptors
b. Lymph node involvement
c. Presence of BRA1 and 2 gene
d. Family history
e. Age

Ans. The key is B. Lymph node involvement. [Risk factors. For breast cancer survivors, factors that increase the risk of a recurrence include: Lymph node involvement. Finding cancer in nearby lymph nodes at the time of your original diagnosis increases your risk of the cancer coming back].

128. A 24-year-old schizophrenic man has recently been depressed. He is on treatment for schizophrenia and depression. Which one of the following increases his risk for suicide?
a. Alcohol
b. Fluoxetine
c. Haloperidol
d.

Advertisement

Carbamazepine
e. Olanzepine

Ans. The key is A. Alcohol. [Alcohol and SSRIs if consumed together has synergistic effect to raise suicidal risk. So alcohol is answer here as the patient is already getting fluoxetine].

129. A man has been diagnosed with indirect inguinal hernia. Which is the single feature of the hernia sac would confirm the diagnosis?
a.    Comes through the femoral ring
b.    Passes through the deep inguinal ring
c.     Lies below the lateral pubic tubercle
d.    Does not pass through the deep inguinal ring

Ans. The key is B. Passes through the deep inguinal ring.

130. A 22-year-old man presented with chronic diarrhea for months. Sygmoidoscopy shows ulcerative strictures. What will be the skin manifestation?
a. Pyoderma gangrenosum
b. Erythema multiforme
c. Necrobiosis lipoidica
d. Erythema chronicum migrans

Ans. The key is A. Pyoderma gangrenosum. [Pyoderma gangrenosum (PG) is the second most common EIM that affects the skin of IBD patients. It seems to affect ulcerative colitis and Crohn's disease (IBD) patients in roughly equal numbers. Fortunately, according to one recent estimate, it affects less than one percent of IBD patients. Most common skin manifestation is erythema nodosum].

131. Each year 950000 peaple develop MI in a population of 250000 million. Out of these, 215000 die and 112500 in the first hour. What is the incidence of MI?
a. 112500 in 215000 million
b. 112500 in 250000 million
c. 215000 in 950000 million
d. 215000 in 250000 million
e. 950000 in 250000 million

Ans. The key is E. 950000 in 250000 million.

132. A 70-year-old man with metastatic cancer is on steroid therapy. His U&E shows sodium 131, Potassium 5.5, and urea 12. What is the most likely underlying condition?
a. Adrenal insufficiency
b. SIADH
c. Hyponatremia
d. Hypernatremia
e. Hypokalemia

Ans. The key is A. Adrenal insufficiency. [Exogenous steroid causes suppression of hypothalamic-pituitary-adrenal axis and any precipitating cause like surgery, infection etc. can lead to adrenal insufficiency. In adrenal insufficiency there is hyponatremia and hyperkalemia].

133. An 8-year-old swallowed 12 tablets of paracetamol 4 hours ago. Serum paracetamol levels when tested were at critical level. What is the single most likely management? 
a. Activated charcoal
b. IV N-acetyl systine
c. Gastric laage
d. Observation only

Ans. The key is B. IV N-acetyl systine.

134. A 79-year-old active man comes to you with complaints of swelling and pain on dorsiflexion of the leg. On examination, there is oedema and a 3 cm increase in diameter of the right cuff on comparison with the left cuff. What is the likely cause?
a. Popliteal artery aneurysm
b. Deep venous thrombosis
c. Popliteal cyst
d. Lymph oedema
e. DMD

Ans. The key is B. Deep venous thrombosis. Swelling of leg, pain on dorsiflexion of foot and increase in diameter of cuff are suggestive of DVT].

135. A pregnant patient who’s Rh negative and has not has been previously sensitized delivers her first baby without complications. What would be the latest time to administer anti-sensitization?
a. 6 hours post partum
b. 24 hours post partum
c. 48 hours post partum
d. 72 hours post partum
e. 5 days post partum

Ans. The key is D. 72 hours post partum.

136. A 64-year-old man complains of increasing shortness of breath and cough for the past 18 months. He cough up a tablespoon of mucopurulent sputum with occasional specks of blood. What is the most likely underlying cause?
a. Acute bronchitis
b. Bronchiectasis
c. Chronic bronchitis
d. Lung cancer
e. Pneumonia

Ans. The key is B. Bronchiectasis. This is a wrong key. Correct key is C. Chronic bronchitis. [In bronchiectasis daily sputum production is cupful not only a tablespoon!! Though 18 month does not fulfill the time requirement to make the diagnosis of chronic bronchitis yet but it is the most likely diagnosis as given info].

137. A 28-year-old man believes that each time the traffic lights change, it means that his mother needs to see him urgently and he needs to return home. He has only slept for 2 hours in the last two days and eats very little. He believes that his thoughts are being taken out of his head and are being made available for the people around him. What is the single most likely diagnosis?
a. Depression
b. GAD
c. Mania
d. Obsessive compulsive disorder
e. Schizophrenia

Ans. E. Schizophrenia. [Delusion, poor sleep, little eating and thought broadcasting are features of schizophrenia].   

138. A 7-year-old girl comes with dysuria and fever of 37.8 C. What is the most appropriate investigation?
a. Clean catch of urine
b. Full blood count
c. Renal ultrasound
d. Suprapubic aspirate

Ans. The key is A. Clean catch of urine. [dysuria, fever points towards UTI for which clean catch of urine is the investigation of choice].

139. A 28-year-old woman who is breast feeding has pain in her left breast. The outer upper quadrant of the breast is red, swollen and tender.  
a. Staphylococcus aureus
b. Staphylococcus epidermidis
c. Streptococcus feacalis
d. Streptococcus pyogenes
e. Streptococcus viridens

Ans. The key is A. Staphylococcus aureus.

140. A 70-year-old man present with sudden onset of severe pain in the lower part of his abdomen. He is unable to pass water and distressed. On examination, the abdomen is distended, tender and dull on percussion. What is the single most appropriate initial management?
a. Arranging abdominal X-ray
b. Giving morphine
c. Inserting urinary catheter
d. Sending blood for amylase
e. Starting IV fluid

Ans. The key is C. Inserting urinary catheter. [A case of acute retention. So urinary catheter should be inserted].

141. A 28-year-old woman suddenly develops a red rash all over the body following a wasp sting. Her pulse is 100 beats/minute, BP 120/70 mmHg, RR 24c/m. What is the single most important clinical feature indicative of adrenaline?
a. IM adrenaline 0.5 ml in 100
b. IM adrenaline 0.5 ml in 1000
c. IM adrenaline 0.5 ml in 10,000
d. IM adrenaline 0.5 ml in 100,000
e. IM adrenaline 0.5 ml in 1,000,000

Ans. The key is B. IM adrenaline 0.5 ml in 1000.

142. A 72-year-old man has had cough and fever for the past 1-week and now has suddenly become short of breath and coughed up a cup full of fresh blood. His temperature is 38.6 C, HR 126 beats/min, saturations 96% on 10L, BP 150/85 mmHg, RR 43c/min. IV cannulation has been done. Which is the most appropriate next step?
a. Antibiotics
b. Arterial blood gas
c. Blood culture
d. Chest X-ray
e. ECG

Ans. The key is A. Antibiotics. [The patient has developed pneumonia. Pneumonia can cause massive hemoptysis and the treatment of pneumonia is antibiotics].

143. A 20-year-old lost control of her car and hit a tree while driving at a speed of 60 mph. She has a large laceration on the parietal scalp and some bruises on the right chest. Her GCS is 14/15 and she is speaking in sentences. She has been given high flow oxygen via facemask. Which is the most immediate management?
a. Cervical spine immobilization
b. Chest tube insertion
c. CT brain scan
d. Endotracheal intubation
e. IV access

Ans. The key is A. Cervical spine immobilization. [Whiplash injury with trauma head. So injury to cervical spine should give urgent importance and cervical spine immobilization should be done early till cervical spinal injury is ruled out].

144. A 35-year-old man skidded on a wet road while riding his motorbike at a speed of 70 mph. He has a large hematoma on the temporal scalp, some bruises on the chest wall and abdomen and a deformed thigh. His GCS is 11/15. He has been given high flow oxygen via facemask. Which is the single most immediate radiological investigation required during the initial resuscitation phage?
a. CT abdomen
b. CT brain
c. X-ray of the chest
d. X-ray of the abdomen
e. X-ray of the femur

Ans. The key is B. CT brain. [35 yrs old man with significant head trauma from RTA with low GCS is highly suggestive of extradural hematoma for which CT brain should be done].

145. A 20-year-old man’s house caught fire. He has blisters and peeling of the skin. He is coughing black sputum. What is the single most appropriate immediate management?
a. Arrange chest X-ray
b. Applying dressing on the face
c. IM analgesia
d. Transfer to burn unit
e. Establish a definitive airway.

Ans. The key is E. Establish a definitive airway. [Coughing black sputum (soot in RTI) indicate inhalation injury and if a burn patient has inhalation injury immediate measurement to be taken is establish a definitive airway].

146. A 50-year-old man present with central crushing chest pain radiating to the left arm. ECG is shown below. What is the next step in management?
a. Warfarine
b. Low molecular weight heparin
c. Percutaneous coronary intervention
d. Aspirin
e. Clopidogrel

Ans. The key is C. Percutaneous coronary intervention. This is wrong key! Correct key is D. Aspirin. [In IHD we give aspirin 300mg stat and then proceed to other procedures].

147. A 60-year-old woman present with post coital bleeding for the past 1 week. What is the most appropriate investigation?
a. Cervical smear
b. Cervical biopsy
c. Colposcopy
d. Endometrial sampling
e. Transvaginal Ultrasound

Ans. The key is C. Colposcopy. [Please note Red Flag Sign for Cancer (either cervical or endometrial) is AGE>35 yrs and persistence of post coital bleeding for more than 4 WEEKS!! So the important clincher here is duration and presenting case still is not supposed to be diagnosed as cancer!! Hence infection is most likely cause or atrophic vaginitis and from the given options C. Colposcopy is the likely correct option].

148. A 2-year-old boy has a temperature of 39.C and purulent otorrhoea. His pinna is laterally and inferiorly displaced. Which is single most appropriate management?
a. Admit for analgesia
b. Admit for intravenous antibiotics
c. Emergency grommet insertion
d. Antibiotic ear drops
e. Oral antibiotics

Ans. The key is B. Admit for intravenous antibiotics. [age and severity of the disease needs iv antibiotics].

149. A 32-year-old woman is having intermittent episodes of vertigo. Each episodes lasts up to 12 hours and is associated with tinnitus and hearing loss. Which is the single medication that may help in reducing the frequency of the vertigo episode?
a. Amitriptyline
b. Paroxetine
c. Cyclizine
d. Betahistine
e. Prochlorperazine

Ans. The key is D. Betahistine. [Case described is Menier’s disease and to reduce the frequency of vertigo episodes (Prophylaxis) betahistine is given].

150. A 72-year-old woman with rheumatoid arthritis has horseness of voice. There is left vocal cord palsy. Which is the joint likely to be involved?
a. Atlanto-occipital
b. Cricoaretinoid
c. Cricothyroid
d. Costrochondral
e. Sternoclavicular

Ans. The key is B. Cricoaretinoid.

151. A 68-year-old woman has otalgia and dysphagia. She has angular cheilitis and pale conjunctiva. Her oropharyngeal examination is normal. In which single anatomical site is this patient likely to have a tumour?
a. Lower oesophagus
b. Nasopharyngeal
c. Post cricoids
d. Thyroids
e. Tonsil

Ans. The key is C. Post cricoid. [Most patients with post cricoid tumour report a globus and dysphagia more to solid than liquids and gradually develops in odynophagia. Progression of tumour causes otalgia which is an ominous sign. Conjunctival pallor is due to tumour related anemia].
 
152. A 4-year-old boy with Down’s syndrome has proven middle ear effusion bilaterally that has been present for 6 months. Which is the single most appropriate management?
a. Adenoidectomy
b. Bilateral mastoidectomy
c. Bilateral grommet ventilation
d. Cochlear implantation
e. Watch and wait for three months

Ans. The key is C. Bilateral grommet ventilation. [Previously grommet insertion was done if fluid persisted beyond three months. Now this concept is changing and degree of hearing loss (a loss of 40 dB or greater is felt to be an absolute indication and a loss in the range of 21-40 dB is a relative indication) is considered for grommet placement].

153. A 20-year-old man has a chronic facial palsy and rinorrhoea.

Advertisement

He has multiple courses of antibiotics with little improvement in his symptoms. A CT scan shows a round opacity with mixed density within the right maxillary sinus. What is the most likely diagnosis?
a. Allergic
b. Angiofibroma
c. Antrochoanal
d. Fungal ball
e. Foreign body within the sinus

Ans. The key is D. Fungal ball. [Failure of multiple courses of antibiotics and CT findings are suggestive of fungal ball].

154. A 78-year-old man has sudden onset of horseness of voice. He has a weak cough and he coughs when eating. He has been a smoker since 18 years ago. He has right sided vocal cord palsy. Which is the single most likely anatomical site of his primary tumour?
a. Bronchus
b. Larynx
c. Oesophagus
d. Oral cavity
e. Parotid

Ans. The key is B. Larynx. [Horseness of voice, weak cough due to vocal cord palsy and smoking makes the likely diagnosis of laryngeal tumour].

155. A 22-year-old woman says she has taken about 40 tablets of paracetamol 3 hours ago. Her heart rate is 110 beats/minute, BP 110/80mmHg, and RR 22c/m. Which is the single most appropriate initial management?
a. Give activated charcoal
b. Give N-acetylcystine
c. Induce vomiting
d. Perform gastric lavage
e. Wait for the 4 hour paracetamol level
 
Ans. The key is E. Wait for the 4 hour paracetamol level. [NICE guideline].

156. A 30-year-old woman says she has taken about 34 tablets of paracetamol about 12 hours ago. She is now unwell. Her heart rate is 120 beats/min, BP 112/78mmHg and RR 20c/m., which is the single most important management?
a. Discuss with the liver transplant unit
b. Give N-acetylcysteine immediately
c. Give activated charcoal immediately
d. Refer for psychiatric assessment
e. Wait for the paracetamol level

Ans. The key is B. Give N-acetylcysteine immediately. [Dangerous dose is 24 tablets and if during 8-15 hours from ingestion start treatment with N-acetylcysteine immediately].

157. A 20-year-old woman says she has taken 40 tablets of paracetamol over the past 3 days. She is not unwell and has vomited a few times. Her heart (probably RR) rate is 22 beats/min. The blood tests are as follows:
Sodium 146mmol/L,
Potassium 3.5mmol/L,
Urea 5.6mmol/L,
Creatinine 115micromol/L,
Albumin 32g/L,
Bilirubin 52micromol/L,
ALT 2560IU/L,
Alkaline phosphatase 230IU/L,
Gamma transpeptidase 230IU/L,
pH 7.3,
INR 4,
Lactate 4.2mmol/L,
Paracetamol 190mg/L.
Which is the single most likely chemical compound responsible for this findings?
a. NAPQI
b. N para-acetyl aminophenol
c. N para-acetyl aminophenol glucoronate
d. Para-acetyl aminophenol sulphate
e. Para-amino phenol

Ans. The key is A. NAPQI. [NAPQI (N-acetyl-p-benzoquinone imine) is a toxic byproduct produced during the xenobiotic metabolism of paracetamol (acetaminophen)].

158. A 36-year-old woman has taken about 28 tablets of paracetamol 6 hours ago. She has been drinking a bottle of vodka and a couple pints of lager everyday for the last few years. She feels nauseous and has vomited a few times.
Sodium 144mmol/L,
Potassium 3.8mmol/L,
Urea 5.6mmol/L,
Creatinine 86micromol/L,
Albumin 30g/L,
Bilirubin 15micromol/L,
ALT 30IU/L,
AST 32IU/L,
Alkaline phosphatase 230IU/L.
Gamma glutamile transpeptidase 1261IU/L,
Paracetamol 80 mg/L

Which is the single most appropriate immediate management?
a. Discussing with liver transplant unit
b. Give acetylcysteine
c. Give fresh frogen plasma
d. Give vitamin K injection
e. Repeat the blood tests

Ans. The key is B. Give acetylcisteine. [Management at 4-8 hours from ingestion: Take drug levels immediately and use graph. As the paracetamol level is high here (80mg/L) give acetylcystine].

159. A 40-year-old man with chronic atrial fibrillation presents with palpitations. What is the most appropriate treatment?
a. Metoprolol
b. Digoxin
c. Verapamil
d. Atenolol
e. Amlodipine

Ans. The key is A. Metoprolol.

160. A 67-year-old man presents with shortness of breath. On examination, he has a harsh pansystolic murmur at the apex which radiates to the axilla. What is the cause.
a. Ventricular septal defect
b. Mitral stenosis
c. Tricuspid regurgitation
d. Mitral stenosis (?)
e. Aortic regurgitation

Ans. The key is C. Tricuspid regurgitation.

161. A healthy baby boy is born at term to a woman who was unwell with confirmed acute hepatitis B during pregnancy. The mother is very concerned that she may have infected the baby with hepatitis B. What is the single preventive measure should be given to the baby?
a.    Full course of hepatitis B vaccine
b.    Hepatitis B immunoglobulin alone
c.     Hepatitis B status testing
d.    Hepatitis B vaccine and hepatitis B immunoglobulin
e.    Hepatitis vaccine as single dose

Ans. The key is D. Hepatitis B vaccine and hepatitis B immunoglobulin.

162. An 18 months girl who has had a single urinary tract infection is seen in the outpatient department. She has fever and vomiting but these improved with a course of trimethoprim. Urine culture became negative. Subsequently a micturating systogram showed bilateral vesico-ureteric reflux. What is the single most appropriate management?
a.    No treatment
b.    Oxybutinin (anti-cholinergic)
c.     Potassium citrate (urinary alkanisation)
d.    Prophylactic antibiotic
e.    Ureteric surgery
 
Ans. The key is E. Ureteric surgery. This is a wrong key! Correct key should be D. Prophylactic antibiotic. [Majority of VUR get cured spontaneously with prophylactic antibiotics within 5 years of age and only a minority needs surgery].        

163. A 6-year-old boy presents with a fever and is unable to swallow his saliva. He has a temperature of 39.C, pulse rate of 130 beats/minute and RR of 40c/m. What is the single most appropriate initial management?
a.    Emergency help from anaesthetist or ENT surgeon
b.    Establish intravenous (IV) access and give IV penicillin
c.     Examine the throat using a tongue depressor
d.    Lay the patient flat and give high flow oxygen
e.    Obtain lateral soft tissue neck x-ray

Ans. The key is A. Emergency help from anaesthetist or ENT surgeon. [Probable dx is acute epiglottitis. Initial management is to establish intubation to save the child from closure of airway which frequently occurs in epiglottitis].

164. A 78-year-old man presents to the A&E department with an upper gastrointestinal hemorrhage. According to his wife, he is taking tablets for osteoporosis to build his bones and volarol for pain relief. The following are causes of gastric or lower oesophageal ulceration except:
a.    Crohn’s disease
b.    Celecoxib
c.     Alendrnate
d.    Misoprostol
e.    Chronic renal failure

Ans. The key is D. Misoprostol. [Misoprostol helps prevent stomach ulcer].

165. A town has population of 500000. In a 5 year period, 100 people present with cancer Y. During the same period, 1250 cases of cancer Y were registered by death certification. And also it was found that in the same city 500 people were found to have cancer Y from the community hospital. The 1-year survival rate is 0%. What is the annual prevalence of cancer Y per million in this population?
a.    200
b.    400
c.     500
d.    700
e.    1250
 
Ans. The key is D. 700. [Probably the question is not correct as the result comes as 740].

166. A 40-year-old patient with malabsorption had biopsy done. On biopsy she was found to large amount of biopsy of lymphocytes. What is the single most likely diagnosis?
a.    Whipples disease = crohn’s disease
b.    Ulcerative disease
c.     Celiac disease = increased lymphocytes
d.    Helicobacter pylori
e.    Gastric cancer

Ans. C. Celiac disease = increased lymphocytes.

167. A 34-year has been drinking for 4 days, now he comes saying that he can feel insect walking on the skin and nasal redness. What is the single most likely drug abuse?
a.    Alcohol
b.    Cocaine
c.     LSD
d.    Heroin
e.    Cannabis

Ans. The key is A. Alcohol.

168. A 40-year-old man presents with abdominal distension 4 days after an open cholecystectomy. On examination there were no bowel sound. What is the diagnosis?
a.    Intestinal obstruction
b.    Paralytic ileus
c.     Internal bleeding
d.    Anastomotic leak
e.    Abdominal aneurysm

Ans. The key is B. Paralytic ileus. [Abdominal distension after surgery with absence of bowel sound suggests paralytic ileus].

169. A 57-year-old man with known diabetes has cholesterol 5.7. Which of the following is the correct combination of medication for his management?
a.    Angiotensin converting enzymes and statin
b.    Angiotensin converting enzyme and metformin
c.     Angiotensin converting enzyme and glibenclamide
d.    Angiotensin converting enzyme and gliclazide
e.    Glibenclamide and statin

Ans. The key is E. Glibenclamide and statin. [This is an incomplete question. Whether we should give sulfonylurea or glibenclamide depends upon BMI. If BMI is <25 we give sulfonylurea and if >25 we give glibenclamide].

170. A 45-year-old man presents to accident and emergency department with headache. He is then found to have high blood pressure of 210/110. His blood results are as follows: Potassium is 2.7, Urea 4.5, creatinine 124, Sodium 136. What is the single most appropriate investigation?
a.    Cortisol
b.    Aldosterone
c.     Adrenaline
d.    Renin
e.    Growth hormone

Ans. The key is B. Aldosterone. [Hypercortisolism can present with similar findings but features like moon face, buffalo hump, etc. makes the differentiating features].

171. An 18-year-old man presents with sudden onset of testicular pain. He has history of multiple sexual partner. Temperature 37.4, blood pressure 120/80. There is no urethral discharge. What is the single most likely diagnosis?
a.    Testicular torsion
b.    Epididymo-orchitis
c.     Testicular tumour
d.    Epididymal cysts
e.    Hydrocele

Ans. The key is A. Testicular torsion. [Sudden onset, normal temperature, and absence of urethral discharge suggests testicular torsion].
 
172. A 45-year-old man presents with ascites and epigastric pain. He had peritoneal lavage done which shows protein less than 25. What is the single most likely cause of ascites?
a.    Tuberculosis
b.    Cirrhosis
c.     Bud chiari syndrome
d.    Alcoholic liver disease
e.    Carcinomatosis

Ans. The key is B. Cirrhosis.  

173. A 65-year-old man presents with ascites. After peritoneal lavage he is found to have low glucose and red cells. What is the single most likely cause?
a.    Gastric cancer
b.    Tuberculosis
c.     Bud chiari syndrome
d.    Liver cirrhosis
e.    Heart failure
 
Ans. The key is A. Gastric cancer. [Both gastric cancer and tuberculosis has the similar picture. TB can occur in any age but cancer usually occurs in elderly].
                                                                                                         
174. A 30-year-old was put on a theater table having an operation with his hand extended. After the operation he is unable to extend his wrist and he also has loss of sensation on the first dorsal area between the thumb and index finger. Which nerve is most likely to be affected?
a.    Radial nerve
b.    Median nerve
c.     Axillary nerve
d.    Musculocutaneous

Ans. The key is A. Radial nerve. [Symptoms (and signs) of radial neuropathy vary depending on the severity of the trauma; however, common symptoms may include wrist drop, numbness (back of the hand and wrist), and inability to voluntarily straighten the fingers. Loss of wrist extension is due to loss of the ability to move of the posterior compartment of forearm muscles].

175. A 51-year-old man was sitting for few hours with his legs crossed. Later he could not dorsiflex or invert is foot. He also has got loss of sensation on the dorsum of his foot. Which nerve is most likely to be damaged?
a.    Peroneal nerve
b.    Sciatic nerve
c.     Sural nerve
d.    Femoral nerve
e.    Tibial nerve

Ans. The key is A. Peroneal nerve. [Symptoms of peroneal nerve injury (foot drop) may include:
i. Inability to point toes toward the body (dorsi flexion)
ii. Pain
iii. Weakness
iv. Numbness (on the shin or top of the foot)
v. Loss of function of foot
vi. High-stepping walk (called steppage gait or footdrop gait)].

176. A mother brings her 2 days old baby worried she could be deaf. The mother is deaf herself and there is family history of deafness as well. What is the single most appropriate investigation?
a. Brainstem evoked response
b. Distraction
c. Tuning fork tests
d. Startle reflex
e. Audiogram

Ans. The key is A. Brainstem evoked response. [Birth to 6 months, Brainstem evoked response].

177. A 45-year-old lady presents with back pain to accident and emergency department. She had a lumbosacral X-ray done which is normal. On examination she is found to have loss of sensation of the lower legs on pin prick. What is the most appropriate investigation?
a. Bone nuclear scan
b. MRI scan
c. Skeletal survey
d. CT scan spine
e. No investigation

Ans. The key is B. MRI scan. [Probable case of disc herniation for which MRI is the investigation of choice].

178. A 50-year-old lady with history of breast cancer treated with chemotherapy and surgery. Now she has come back with pelvic pain. What is the most appropriate management?
a. DEXA
b. Radiotherapy
c. Additional chemotherapy
d. Bisphosphonates
e. Mamography

Ans. The key is B. Radiotherapy. [External beam radiotherapy and systemic endocrine and cytotoxic treatments are the mainstay of treatment in advanced cancers. However, it is now clear that the bisphosphonates provide an additional treatment strategy, which reduces both the symptoms and complications of bone involvement].

179. A 2-year-old child was found to have meningitis. The child recovered well. He was not resuscitated. What would be the expected outcome of the child?
a. Goes back to normal status like before the illness
b. Hearing loss
c. Visual loss
d. Mental state retardation
e. Hemiperasis

Ans. The key is A. a. Goes back to normal status like before the illness.

180. A 35-year-old man presents with headache and rash. On examination the rash does not blanch. What is the single most appropriate antibiotics?
a. Cefotaxime
b. Amoxicillin
c. Ciprofloxacin
d. Vancomycin
e. Clarithromycine

Ans. The key is  A. Cefotaxime. [The likely diagnosis is meningococcal septicemia which is treated with Cefotaxime (OHCM, 9th edition, page 833)].

181. A 45-year-old man presents with fever, rash and headache. His Glasgow coma scale is 10/15. What is the single most appropriate immediate management?
a. Intravenous antibiotics and CT scan brain
b. Lumber puncture and IV antibiotics
c. CT scan of the brain and intravenous fluids
d. Lumber puncture and antibiotics
e. Blood culture and lumber puncture

Ans. The key is A. Intravenous antibiotics and CT scan brain.

182. A 47-year-old man presents with fever, headache, neck stiffness and photophobia. He is also vomiting. What is the single most diagnostic investigation?
a. Blood culture
b. CT scan brain
c. Blood culture
d. Lumber puncture
e. MRI brain

Ans. The key is D. Lumber puncture. [Though vomiting is a warning sign of possible raised intracranial pressure but it is used as a distraction here. The question is not next or initial investigation but the SINGLE MOST DIAGNOSTIC investigation which is lumber puncture. So for this question the answer is D. Lumber puncture.].

183.

Advertisement

A 40-year-old man presents to accident and emergency department with confusion, ataxia and ophthalmoplegia. He lost his job recently and he is smelling of sweet breath. Alcohol breath test is negative. What is the single most appropriate immediate management?
a. Glucose
b. Intravenous fluid
c. Intravenous thiamine
d. Potassium chloride
e. Chlordiazepoxide

Ans. The key is C. Intravenous thiamine. [Classic triad of Wernicke’s encephalopathy. Treated with intravenous thiamine].

184. A 40-year-old man lost his job 1 year ago. The police picked him from the street. He has a sweet smell. Alcohol breath test was negative. What is the most likely diagnosis?
a.    Alohol intoxication
b.    Alcohol withdrawal
c.     Delirium tremens
d.    Wernicke’s encephalopathy
e.    Acute psychosis

Ans. The key is D. Wernicke’s encephalopathy. [As alcohol breath test is negative it is not alcohol intoxicatin and police picked him and done alcohol breath test supports wernicke’s encephalopathy (ataxia probably was noticed)].

185. An 18-year-old lady was found to have BMI of 22. She had lost 8kg of weight in 6 months. She has low mood, she sad all the time. She then started to buy expensive clothes and eating in expensive restaurant. What is the single most appropriate diagnosis?
a.    Anorexia nervosa
b.    Bulaemia nervosa
c.     Bipolar affective disorder
d.    Schizophrenia
e.    Depression

Ans. The key is C. Bipolar affective disorder.

186. A 3-yr-old girl fell down while riding her bicycle. She then went pale and then rigid. 2 minutes later she recovered fully but she was still pale. After 1 hour she well and playing around. What is the most appropriate management?
a.    No investigations
b.    Electrocardiogram
c.     Full blood count
d.    Electroencephalogram
e.    CT scan of the brain

Ans. The key is A. No investigations. [It is breath-holding spell which is transient and becomes normal within few minutes. No investigation is needed].

187. A 19-year-old man does not talk to anyone. He has collected 2000 toy cars. He is indifferent to his mum. What is the single most appropriate investigation?
a.    Autistic spectrum disorder
b.    Psychosis
c.     Borderline personality disorder
d.    Dissociative disorder
e.    Schizotypal disorder

Ans. The key is A. Autistic spectrum disorder.

188. A 30-year-old lady every time she leaves the house experiences palpitation and difficulty in breathing. She has to rush back into her house to allay her anxiety with a drink. What is the single most likely diagnosis?
a.    Social phobia
b.    Agarophobia
c.     Panic attack
d.    Hysteria
e.    Arachnophobia

Ans. The key is B. Agarophobia. [Agoraphobia is a fear of being in situations where escape might be difficult or that help wouldn't be available if things go wrong.
Many people assume agoraphobia is simply a fear of open spaces, but it's actually a more complex condition. Someone with agoraphobia may be scared of:
i. Travelling on public transport
ii. Visiting a shopping centre
iii. Leaving home].

189. A 40-year-old lady was biten by a bee on her right hand 3 weeks ago. She has past history of past medical history breast cancer for which she had radiotherapy and surgery with axillary clearance. She presents to accident and emergency department with swelling of the right hand. What is the single most likely diagnosis?
a.    Anaphylaxis
b.    Delayed allergic reaction
c.     Cellulitis
d.    Lymphoedema
e.    Superior vena caval obstruction

Ans. The key is D. Lymphoedema. [axillary clearance may cause compromised lymphatic drainage of upper limb leading to lymphoedema].

190. A 60-year-old lady who had anterior resection of his bowel 5 days ago presents with swelling on her wound. There is 4cm swelling is around the colostomy which tender and it fluctuating. She is not systematically unwell. What is the single most appropriate action?
a.    Laparotomy
b.    Local anesthesia
c.     Laparotomy with wound repair
d.    Exploration of the wound
e.    Abdominal X-ray

Ans. The key is D. Exploration of the wound.    

191. A 25 year old lady presents to outpatient department requesting contraception. She has irregular and painful periods. She smokes 20 cigarettes a day. Otherwise she is fit and well. Which is the single most appropriate type of contraception?
a.    Combined oral contraception
b.    Progesterone only contraception
c.     Trenexamic acid
d.    Levenogestrel
e.    Levanogestrel IUCD

Ans. A. Combined oral contraception [smoking >20 cigarettes at age >35yrs are contraindication for COCP, In irregular period COCP is the drug of choice as this patient is <35 best choice for her is A. Combined oral contraception].

192. A 16-year-old girl presents with heavy regular periods. What is the single most appropriate treatment?
a.    Combined oral contraception
b.    Progesterone only pill
c.     Trenexamic acid
d.    Mefenamic acid
e.    Levonogestrel intrauterine system

Ans. The key is C. Trenexamic acid. [As the patient did not ask for contraception or not described as sexually active, trenexamic acid is best option for her].

193. A 30-year-old lady with past medical history of pulmonary embolism presents to outpatient clinic requesting contraception.  She has irregular and painful periods. What is the single most appropriate contraception?
a.    Intrauterine contraceptive device
b.    Combined pill contraception
c.     Progesterone only pill
d.    Mefenamic acid
e.    Trenexamic acid

Ans. The key is C. Progesterone only pill. [If irregular period COCP is the contraception of choice. But as there is history of pulmonary embolism COCP should be avoided and next option is progesterone only pill].

194. A 34-year-old man had a 4mm ureteric stone which he passed with urine. This time he presents with 3cm stone in right kidney. What is the single most appropriate treatment?
a.    No treatment
b.    Lithotripsy-shock wave
c.     Laparotomy
d.    Operative stone removal
e.    Observe

Ans. The key is E. Observe. This is wrong key. Correct key is D. Operative stone removal. [kidney stone is risk factor for infection and renal damage. So there is no scope for observation but removal].

195. A 35-year-old man returned from Egypt where she was working in outdoor. She was found to have a 5cm stone which was removed operative. She is now planning to go back to Egypt. What advice would you give her?
a.    Increase fluid intake
b.    Better not to go to Egypt again
c.     Reduce fluid intake
d.    Regular hospital admission for intravenous fluids
e.    Return to UK regularly

Ans. The key is A. Increase fluid intake.

196. A 28-year-old man with a known psychiatric disease thinks that he is the professor of medicine and he says later he will become God. What is the single most likely diagnosis?
a.    Delusion of grandiose
b.    Idea of reference
c.     Identity disorder
d.    Paranoid delusion
e.    Delusion of reference

Ans. The key is A. Delusion of grandiose. [1. Delusion of grandiose- A delusion of grandiose is the fixed, false belief that one possesses superior qualities such as genius, fame, omnipotence, or wealth. 2. Idea of reference- The patient falsely believe that everything occurring around them is related somehow to them when in fact, it isn't. 3. Identity disorder- Condition in which two or more distinct identities, or personality states, are present in—and alternately take control of—an individual. Paranoid delusion- Delusional disorder is a mental illness in which the patient presents with delusions, but with no accompanying prominent hallucinations, thought disorder, mood disorder, or significant flattening of affect. 5. Delusion of reference- synonymous to Idea of reference described earlier].

197. A 30-year-old lady with past history of 1 episode of mania presents with depression. She has been treated successfully this time but she needs prophylaxis for her condition. What is the most appropriate medication/
a.    Mood stabilizer
b.    Benzodiazepines
c.     Anti-depressant
d.    Anti-psychotic
e.    Anti-epileptic

Ans. The key is A. Mood stabilizer.

198. A 35-year-old diabetic patient was on long acting anti diabetic in the morning and short acting in the evening. After lunch patient gets hypoglycemia. How you will manage this patient?
a.    Provide more food at lunch
b.    Decrease long acting in the morning
c.     Decrease short acting in the afternoon
d.    Decrease both dose
e.    Reassurance

Ans. The key is B. Decrease long acting in the morning.

199. A 70-year-old man with lung cancer presents with a cough. He is already on codeine, steroid and salbutamol inhalers. What is the single most appropriate management?
a.    Nebulized adrenaline
b.    0.9% saline nebulized
c.     Oral morphine
d.    Antibiotics
e.    Benzodiazepines

Ans. The key is B. 0.9% saline nebulized. [Nebulized NS loosens the phlem and help to expectorate it easily].

200. A 65-year-old man presented with shortness of breath. He was a shipyard worker. He was admitted treated with antibiotics and inhalers. What would be the reason to inform the coroner?
a.    Industrial disease
b.    Inappropriate treatment
c.     Because he died during admission
d.    To satisfy the relatives
e.    It should be done as a routine

Ans. The key is a. Industrial disease. [The coroner is an independent judicial officer. The coroner's job is to establish why the person has died and the surrounding circumstances. The coroner's involvement is unrelated to any claim for compensation. However, a coroner's verdict of 'industrial disease', as the cause of death, can be used to support a claim]..

Was my post useful? Support me to keep creating useful content

Disclaimer If this post is your copyrighted property, please message this user or email us your request at team@pejoweb.com with a link to this post




3 likes
 

Advertisement

";

Advertisement